¿Es el mecanismo de Higgs una transformación de calibre o no? (U(1)U(1)U(1) contexto)

Estoy tratando de entender la forma en que se aplica el mecanismo de Higgs en el contexto de un tu ( 1 ) escenario de ruptura de simetría, lo que significa que tengo un campo complejo de Higgs ϕ = mi i ξ ( ρ + v ) 2 y quiero medir el ξ campo que está causando mi término fuera de la diagonal, en ruptura de simetría normal. Presento las siguientes reglas de transformación que se cumplen para preservar la invariancia de calibre local en la simetría espontánea que rompe no 0 vev para ρ :

{ ϕ mi i θ ϕ A m A m 1 q m θ D m = m + i q A m

Según tengo entendido, el mecanismo de fijación del calibre de Higgs se utiliza para especificar las transformaciones que calibran ξ lejos. La idea es que queremos buscar el ángulo que nos da un campo de Higgs con un grado real de libertad, como en

ϕ mi i θ ϕ = mi i θ mi i ξ ( ρ + v ) 2 = mi i θ ( ρ + v ) 2

entonces

{ ϕ mi i θ ϕ = mi i θ mi i ξ ( ρ + v ) 2 = mi i θ ( ρ + v ) 2 A m A m 1 q m θ

dónde θ = θ + ξ . Omito algunos factores de v en la exponencial por el momento. Eso es lo que veo que hacen mis libros. Para θ = 0 esto se convierte

{ ϕ ( ρ + v ) 2 A m A m
y el resto son las interacciones deseadas derivadas y los términos en general. Observo que esto no preserva la invariancia del calibre local porque

{ ϕ mi i θ ( ρ + v ) 2 mi i θ ϕ A m A m 1 q m θ
Entonces, ¿esta transformación es la que hacemos o me he equivocado en alguna parte y se puede hacer correctamente a través de una transformación de calibre legítima?

¡Hola Karky, y bienvenido a Physics Stack Exchange! Esta es una buena pregunta tal como está, pero creo que un par de cosas impiden que sea una gran pregunta: primero, "¿Estoy viendo esto de la manera correcta?" es un poco vago. ¿De qué otra manera crees que podrías verlo, o qué te hace pensar exactamente que la forma en que lo estás viendo ahora podría no ser válida? Además, "Pregunta del mecanismo Higgs U (1)" no es un título muy bueno. Si aborda lo primero, probablemente sugerirá un mejor título. Tenemos algunos consejos para escribir buenos títulos .
Gracias por las sugerencias, modifiqué el título y la pregunta para ser más específicos.
no entiendo tu pregunta Cuando tengas θ = 0 , has medido exactamente el ξ fase que estaba allí antes, es decir, la transformación de calibre ϕ mi i ξ ϕ te deja con mi i ξ ϕ = 1 2 ( ρ + v ) . No te transformas por θ , te transformas con θ , y para θ = ξ , obtendrá el formulario deseado.
@ACuriousMind parece que podría ser una respuesta
@ACuriousMind Lo que me confunde es el hecho de que si te transformas como
{ ϕ mi i ξ ϕ A m A m 1 q m ( ξ ) = A m + 1 q m ξ
entonces el Lagrangiano, como invariante bajo estas transformaciones, retiene su forma original para el campo ϕ , pero que eso no es lo que queremos ya que entonces el Lagrangiano tiene una exponencial de ξ y el campo no está medido.
Si absorbimos la exponencial en ϕ , para obtener solo el campo real, entonces el 1 q m ξ término se multiplica en el término cinético y nos deja con ξ términos por todas partes al final.

Respuestas (3)

No, lo hiciste bien. Usted se transformó en calibre al llamado calibre unitario donde las 2 partículas del doblete complejo φ ahora se han reducido solo a un componente real ρ del mismo, mientras que el componente de fase se ha mutado en un componente del campo de calibre.

{ ϕ ( ρ + v ) 2 A m A m
donde "el resto" incluye un término de masa invariante que no es de calibre para el campo de calibre , por lo que ahora tiene 3 componentes, no 2 debido a eso: θ ' ha sido reasignado, a través de una transformación de calibre, del escalar complejo al campo de calibre .

Ahora se encuentra en un calibre determinado: si pasara a otro calibre, no vería estos grados de libertad tan simplemente, pero, por supuesto, los resultados para las amplitudes (invariantes del calibre) que calcularía serían idénticos.

Ese es el punto: la invariancia de calibre permite la transición a un calibre donde el contenido físico de la teoría es más transparente.

Solo para agregar a la respuesta de Cosmas. Creo que la gente confunde la transformación de calibre y la fijación de calibre . Lo que estamos haciendo aquí es fijar el calibre, lo que significa que restringimos los campos de una elección arbitraria. ( ϕ C , A m ) a una opción fija de calibre ( ϕ 0 R , A m ) . El punto de hacer esto es que eliminamos la redundancia debido a la simetría de calibre, en el sentido de que cada punto ( ϕ C , A m ) se puede alcanzar exactamente una vez haciendo una transformación de calibre en el espacio de campo restringido (calibre fijo). Más específicamente, supongamos que queremos llegar al punto ( ϕ 0 mi i ξ , A m ) (dónde ϕ 0 R ), debemos comenzar desde el punto fijo de calibre ( ϕ 0 , A m + 1 q m ξ ) luego haga la transformación de calibre con el parámetro de transformación θ = ξ (siguiendo convención de @karky). En resumen, al eliminar el bosón de Goldstone ξ en el modelo abeliano de Higgs, lo que estamos haciendo es fijar el calibre, NO transformar el calibre. Cualquier transformación de calibre deja el lagrangiano invariante, por lo que no hay esperanza de que el campo ξ puede ser eliminado.

Introduzcamos un poco más de notación porque creo que te estás confundiendo con el notación:

Dejar θ : R 4 R ser cualquier función. Entonces los campos transformados de calibre son

ϕ θ := mi i θ ϕ A θ := A 1 q d θ
y una transformación de calibre por θ está reemplazando ϕ , A por ϕ θ , A θ , que es lo que denotas por ϕ ϕ θ , A A θ .

La invariancia de calibre significa que S [ ϕ , A ] = S [ ϕ θ , A θ ] , es decir, la acción no cambia bajo una transformación de indicador, y en este caso también significa L [ ϕ , A ] = L [ ϕ θ , A θ ] . 1 Esto es válido para este Lagrangiano en un nivel abstracto.

Ahora, sabes que puedes escribir ϕ = mi i ξ ϕ 0 , y quiere expresar el Lagrangiano puramente en términos de ϕ 0 sin tener el ξ ahí. Esto se logra fijando el manómetro para que sea θ = ξ , desde ϕ ξ = ϕ 0 . Dado que la teoría es invariante de calibre, tiene L [ ϕ , A ] = L [ ϕ ξ , A ξ ] .

Su problema principal parece ser que el lhs parece contener todavía ξ porque el A ξ contiene ξ . Eso es de hecho un problema que necesita ser arreglado. Hay dos formas de hacer esto, solo daré la breve por ahora, que desafortunadamente presupone ξ ser infinitesimal:

Tenga en cuenta que para infinitesimal ξ y ρ , ϕ = ϕ 0 + i v ξ . Ahora calcule el Lagrangiano L [ ϕ 0 + i v ξ , A ] y observa que obtienes un término que se parece a 2 1 2 mi 2 v 2 ( A 1 q d ξ ) 2 . Por lo tanto, enchufar A ξ de hecho mata a la ξ también en los términos con A , ya que esto transforma el término entre paréntesis en A 2 sin ningún ξ .


1 En general, el Lagrangiano puede cambiar por una derivada total

2>/sup>"Parece" porque hay varias convenciones diferentes sobre cómo se ve exactamente la transformación de calibre y la expansión alrededor del VEV, y no me inclino a rastrear cuál está en uso aquí.

Quieres decir L [ ϕ 0 + i v ξ , A ] en la tercera línea desde el final?
@karky: Sí, corregido.
Si pregunto por la forma no infinitesimal, ¿podría darme una referencia?
@karky: Busqué, pero no pude encontrar uno. Tengo una idea aproximada de cómo se supone que funciona, pero necesitaría resolverlo en detalle; Lo agregaré a esta respuesta cuando encuentre el tiempo (o una referencia).
Ahora veo lo que estaba pensando mal. Usando su notación, estaba tratando de expresar el Lagrangiano transformado [ ϕ θ , A ] donde en cambio debería estar usando [ ϕ θ , A θ ] . Específicamente, estaba expresando A θ con respecto a A pero usó la versión transformada de ϕ . Perdí demasiado tiempo en este malentendido, ¡así que gracias por su ayuda catalítica!